Difference between revisions of "2006 AMC 12A Problems/Problem 8"

m (Solution: ??)
m (wikified)
Line 1: Line 1:
 
== Problem ==
 
== Problem ==
  
How many sets of two or more consecutive positive integers have a sum of <math>15</math>?
+
How many [[set]]s of two or more consecutive [[positive integer]]s have a sum of <math>15</math>?
  
 
<math> \mathrm{(A) \ } 1\qquad \mathrm{(B) \ } 2\qquad \mathrm{(C) \ } 3\qquad \mathrm{(D) \ } 4\qquad \mathrm{(E) \ }  5</math>
 
<math> \mathrm{(A) \ } 1\qquad \mathrm{(B) \ } 2\qquad \mathrm{(C) \ } 3\qquad \mathrm{(D) \ } 4\qquad \mathrm{(E) \ }  5</math>
  
 
== Solution ==
 
== Solution ==
Notice that if the consecutive positive integers have a sum of 15, then their average (which could be a fraction) must be a factor of 15. If the number of integers in the list is odd, then the average must be either 1, 3, or 5, and 1 is clearly not possible. The other two possibilities both work:
+
Notice that if the consecutive positive integers have a sum of 15, then their [[average]] (which could be a [[fraction]]) must be a [[divisor]] of 15. If the number of [[integer]]s in the list is [[odd]], then the average must be either 1, 3, or 5, and 1 is clearly not possible. The other two possibilities both work:
  
 
*<math>1 + 2 + 3 + 4 + 5 = 15</math>
 
*<math>1 + 2 + 3 + 4 + 5 = 15</math>
Line 15: Line 15:
 
*<math>15 = 7 + 8</math>
 
*<math>15 = 7 + 8</math>
  
Thus, the correct answer is C (3).
+
Thus, the correct answer is 3, answer choice <math>\mathrm{(C) \ }</math>.
  
 
== See also ==
 
== See also ==

Revision as of 23:14, 30 January 2007

Problem

How many sets of two or more consecutive positive integers have a sum of $15$?

$\mathrm{(A) \ } 1\qquad \mathrm{(B) \ } 2\qquad \mathrm{(C) \ } 3\qquad \mathrm{(D) \ } 4\qquad \mathrm{(E) \ }  5$

Solution

Notice that if the consecutive positive integers have a sum of 15, then their average (which could be a fraction) must be a divisor of 15. If the number of integers in the list is odd, then the average must be either 1, 3, or 5, and 1 is clearly not possible. The other two possibilities both work:

  • $1 + 2 + 3 + 4 + 5 = 15$
  • $4 + 5 + 6 = 15$

If the number of integers in the list is even, then the average will have a $\frac{1}{2}$. The only possibility is $\frac{15}{2}$, from which we get:

  • $15 = 7 + 8$

Thus, the correct answer is 3, answer choice $\mathrm{(C) \ }$.

See also


{{{header}}}
Preceded by
Problem 7
AMC 12A
2006
Followed by
Problem 9